n-th order polynomial with all roots where all coefficients are 1 or -1, highest order of n?
$begingroup$
I have polynomial $f(x) = sum_{i=0}^n a_i x^i $, where $a_i = pm 1$. All roots of $f(x)$ are real. What's the highest order of $n$? Note that the roots are real but can be irrational. Even if there are duplicate roots, that's fine
number-theory polynomials
$endgroup$
add a comment |
$begingroup$
I have polynomial $f(x) = sum_{i=0}^n a_i x^i $, where $a_i = pm 1$. All roots of $f(x)$ are real. What's the highest order of $n$? Note that the roots are real but can be irrational. Even if there are duplicate roots, that's fine
number-theory polynomials
$endgroup$
$begingroup$
I assume you are not counting multiplicity of the roots (i.e. the n roots are all distinct).
$endgroup$
– D.B.
Dec 25 '18 at 4:02
$begingroup$
@D.B. yes, even if there are duplicate roots, that's fine
$endgroup$
– CodeNoob
Dec 25 '18 at 4:04
1
$begingroup$
An example with $n=3$ is $x^3-x^2-x+1 = (x-1)^2(x+1)$. There don't seem to be any for $n=4, 5$, $6$ or $7$.
$endgroup$
– Robert Israel
Dec 25 '18 at 4:15
$begingroup$
@RobertIsrael that's right. We can prove all roots have abs value between (0.5,2) as well.
$endgroup$
– CodeNoob
Dec 25 '18 at 4:40
$begingroup$
Descartes’ Rule of Signs might be helpful here.
$endgroup$
– Clayton
Dec 25 '18 at 4:49
add a comment |
$begingroup$
I have polynomial $f(x) = sum_{i=0}^n a_i x^i $, where $a_i = pm 1$. All roots of $f(x)$ are real. What's the highest order of $n$? Note that the roots are real but can be irrational. Even if there are duplicate roots, that's fine
number-theory polynomials
$endgroup$
I have polynomial $f(x) = sum_{i=0}^n a_i x^i $, where $a_i = pm 1$. All roots of $f(x)$ are real. What's the highest order of $n$? Note that the roots are real but can be irrational. Even if there are duplicate roots, that's fine
number-theory polynomials
number-theory polynomials
edited Dec 25 '18 at 14:37
CodeNoob
asked Dec 25 '18 at 3:45
CodeNoobCodeNoob
23019
23019
$begingroup$
I assume you are not counting multiplicity of the roots (i.e. the n roots are all distinct).
$endgroup$
– D.B.
Dec 25 '18 at 4:02
$begingroup$
@D.B. yes, even if there are duplicate roots, that's fine
$endgroup$
– CodeNoob
Dec 25 '18 at 4:04
1
$begingroup$
An example with $n=3$ is $x^3-x^2-x+1 = (x-1)^2(x+1)$. There don't seem to be any for $n=4, 5$, $6$ or $7$.
$endgroup$
– Robert Israel
Dec 25 '18 at 4:15
$begingroup$
@RobertIsrael that's right. We can prove all roots have abs value between (0.5,2) as well.
$endgroup$
– CodeNoob
Dec 25 '18 at 4:40
$begingroup$
Descartes’ Rule of Signs might be helpful here.
$endgroup$
– Clayton
Dec 25 '18 at 4:49
add a comment |
$begingroup$
I assume you are not counting multiplicity of the roots (i.e. the n roots are all distinct).
$endgroup$
– D.B.
Dec 25 '18 at 4:02
$begingroup$
@D.B. yes, even if there are duplicate roots, that's fine
$endgroup$
– CodeNoob
Dec 25 '18 at 4:04
1
$begingroup$
An example with $n=3$ is $x^3-x^2-x+1 = (x-1)^2(x+1)$. There don't seem to be any for $n=4, 5$, $6$ or $7$.
$endgroup$
– Robert Israel
Dec 25 '18 at 4:15
$begingroup$
@RobertIsrael that's right. We can prove all roots have abs value between (0.5,2) as well.
$endgroup$
– CodeNoob
Dec 25 '18 at 4:40
$begingroup$
Descartes’ Rule of Signs might be helpful here.
$endgroup$
– Clayton
Dec 25 '18 at 4:49
$begingroup$
I assume you are not counting multiplicity of the roots (i.e. the n roots are all distinct).
$endgroup$
– D.B.
Dec 25 '18 at 4:02
$begingroup$
I assume you are not counting multiplicity of the roots (i.e. the n roots are all distinct).
$endgroup$
– D.B.
Dec 25 '18 at 4:02
$begingroup$
@D.B. yes, even if there are duplicate roots, that's fine
$endgroup$
– CodeNoob
Dec 25 '18 at 4:04
$begingroup$
@D.B. yes, even if there are duplicate roots, that's fine
$endgroup$
– CodeNoob
Dec 25 '18 at 4:04
1
1
$begingroup$
An example with $n=3$ is $x^3-x^2-x+1 = (x-1)^2(x+1)$. There don't seem to be any for $n=4, 5$, $6$ or $7$.
$endgroup$
– Robert Israel
Dec 25 '18 at 4:15
$begingroup$
An example with $n=3$ is $x^3-x^2-x+1 = (x-1)^2(x+1)$. There don't seem to be any for $n=4, 5$, $6$ or $7$.
$endgroup$
– Robert Israel
Dec 25 '18 at 4:15
$begingroup$
@RobertIsrael that's right. We can prove all roots have abs value between (0.5,2) as well.
$endgroup$
– CodeNoob
Dec 25 '18 at 4:40
$begingroup$
@RobertIsrael that's right. We can prove all roots have abs value between (0.5,2) as well.
$endgroup$
– CodeNoob
Dec 25 '18 at 4:40
$begingroup$
Descartes’ Rule of Signs might be helpful here.
$endgroup$
– Clayton
Dec 25 '18 at 4:49
$begingroup$
Descartes’ Rule of Signs might be helpful here.
$endgroup$
– Clayton
Dec 25 '18 at 4:49
add a comment |
1 Answer
1
active
oldest
votes
$begingroup$
Vieta's Formulas are the key to this problem. Let $r_1, cdots, r_n$ be the roots. Then define
begin{align}
A &= sum_{i=1}^n r_i \
B &= sum_{1 le i_2 < i_2 le n} r_{i_1}r_{i_2} \
C &= prod_{i=1}^n r_i .
end{align}
By Vieta's Formulas, we know that $A, B, C in {pm 1}.$ Now
$$sum_{i=1}^n r_i^2 = A^2 - 2B = 1-2B ge 0 implies B = -1.$$
But then by AM-GM, we have
$$frac{3}n = frac{1}n sum_{i=1}^n r_i^2 ge left(C^2 right)^{1/n} = 1 $$
which cannot happen for $n ge 4$.
$endgroup$
add a comment |
Your Answer
StackExchange.ready(function() {
var channelOptions = {
tags: "".split(" "),
id: "69"
};
initTagRenderer("".split(" "), "".split(" "), channelOptions);
StackExchange.using("externalEditor", function() {
// Have to fire editor after snippets, if snippets enabled
if (StackExchange.settings.snippets.snippetsEnabled) {
StackExchange.using("snippets", function() {
createEditor();
});
}
else {
createEditor();
}
});
function createEditor() {
StackExchange.prepareEditor({
heartbeatType: 'answer',
autoActivateHeartbeat: false,
convertImagesToLinks: true,
noModals: true,
showLowRepImageUploadWarning: true,
reputationToPostImages: 10,
bindNavPrevention: true,
postfix: "",
imageUploader: {
brandingHtml: "Powered by u003ca class="icon-imgur-white" href="https://imgur.com/"u003eu003c/au003e",
contentPolicyHtml: "User contributions licensed under u003ca href="https://creativecommons.org/licenses/by-sa/3.0/"u003ecc by-sa 3.0 with attribution requiredu003c/au003e u003ca href="https://stackoverflow.com/legal/content-policy"u003e(content policy)u003c/au003e",
allowUrls: true
},
noCode: true, onDemand: true,
discardSelector: ".discard-answer"
,immediatelyShowMarkdownHelp:true
});
}
});
Sign up or log in
StackExchange.ready(function () {
StackExchange.helpers.onClickDraftSave('#login-link');
});
Sign up using Google
Sign up using Facebook
Sign up using Email and Password
Post as a guest
Required, but never shown
StackExchange.ready(
function () {
StackExchange.openid.initPostLogin('.new-post-login', 'https%3a%2f%2fmath.stackexchange.com%2fquestions%2f3051827%2fn-th-order-polynomial-with-all-roots-where-all-coefficients-are-1-or-1-highest%23new-answer', 'question_page');
}
);
Post as a guest
Required, but never shown
1 Answer
1
active
oldest
votes
1 Answer
1
active
oldest
votes
active
oldest
votes
active
oldest
votes
$begingroup$
Vieta's Formulas are the key to this problem. Let $r_1, cdots, r_n$ be the roots. Then define
begin{align}
A &= sum_{i=1}^n r_i \
B &= sum_{1 le i_2 < i_2 le n} r_{i_1}r_{i_2} \
C &= prod_{i=1}^n r_i .
end{align}
By Vieta's Formulas, we know that $A, B, C in {pm 1}.$ Now
$$sum_{i=1}^n r_i^2 = A^2 - 2B = 1-2B ge 0 implies B = -1.$$
But then by AM-GM, we have
$$frac{3}n = frac{1}n sum_{i=1}^n r_i^2 ge left(C^2 right)^{1/n} = 1 $$
which cannot happen for $n ge 4$.
$endgroup$
add a comment |
$begingroup$
Vieta's Formulas are the key to this problem. Let $r_1, cdots, r_n$ be the roots. Then define
begin{align}
A &= sum_{i=1}^n r_i \
B &= sum_{1 le i_2 < i_2 le n} r_{i_1}r_{i_2} \
C &= prod_{i=1}^n r_i .
end{align}
By Vieta's Formulas, we know that $A, B, C in {pm 1}.$ Now
$$sum_{i=1}^n r_i^2 = A^2 - 2B = 1-2B ge 0 implies B = -1.$$
But then by AM-GM, we have
$$frac{3}n = frac{1}n sum_{i=1}^n r_i^2 ge left(C^2 right)^{1/n} = 1 $$
which cannot happen for $n ge 4$.
$endgroup$
add a comment |
$begingroup$
Vieta's Formulas are the key to this problem. Let $r_1, cdots, r_n$ be the roots. Then define
begin{align}
A &= sum_{i=1}^n r_i \
B &= sum_{1 le i_2 < i_2 le n} r_{i_1}r_{i_2} \
C &= prod_{i=1}^n r_i .
end{align}
By Vieta's Formulas, we know that $A, B, C in {pm 1}.$ Now
$$sum_{i=1}^n r_i^2 = A^2 - 2B = 1-2B ge 0 implies B = -1.$$
But then by AM-GM, we have
$$frac{3}n = frac{1}n sum_{i=1}^n r_i^2 ge left(C^2 right)^{1/n} = 1 $$
which cannot happen for $n ge 4$.
$endgroup$
Vieta's Formulas are the key to this problem. Let $r_1, cdots, r_n$ be the roots. Then define
begin{align}
A &= sum_{i=1}^n r_i \
B &= sum_{1 le i_2 < i_2 le n} r_{i_1}r_{i_2} \
C &= prod_{i=1}^n r_i .
end{align}
By Vieta's Formulas, we know that $A, B, C in {pm 1}.$ Now
$$sum_{i=1}^n r_i^2 = A^2 - 2B = 1-2B ge 0 implies B = -1.$$
But then by AM-GM, we have
$$frac{3}n = frac{1}n sum_{i=1}^n r_i^2 ge left(C^2 right)^{1/n} = 1 $$
which cannot happen for $n ge 4$.
answered Dec 25 '18 at 5:15
Sandeep SilwalSandeep Silwal
5,91811337
5,91811337
add a comment |
add a comment |
Thanks for contributing an answer to Mathematics Stack Exchange!
- Please be sure to answer the question. Provide details and share your research!
But avoid …
- Asking for help, clarification, or responding to other answers.
- Making statements based on opinion; back them up with references or personal experience.
Use MathJax to format equations. MathJax reference.
To learn more, see our tips on writing great answers.
Sign up or log in
StackExchange.ready(function () {
StackExchange.helpers.onClickDraftSave('#login-link');
});
Sign up using Google
Sign up using Facebook
Sign up using Email and Password
Post as a guest
Required, but never shown
StackExchange.ready(
function () {
StackExchange.openid.initPostLogin('.new-post-login', 'https%3a%2f%2fmath.stackexchange.com%2fquestions%2f3051827%2fn-th-order-polynomial-with-all-roots-where-all-coefficients-are-1-or-1-highest%23new-answer', 'question_page');
}
);
Post as a guest
Required, but never shown
Sign up or log in
StackExchange.ready(function () {
StackExchange.helpers.onClickDraftSave('#login-link');
});
Sign up using Google
Sign up using Facebook
Sign up using Email and Password
Post as a guest
Required, but never shown
Sign up or log in
StackExchange.ready(function () {
StackExchange.helpers.onClickDraftSave('#login-link');
});
Sign up using Google
Sign up using Facebook
Sign up using Email and Password
Post as a guest
Required, but never shown
Sign up or log in
StackExchange.ready(function () {
StackExchange.helpers.onClickDraftSave('#login-link');
});
Sign up using Google
Sign up using Facebook
Sign up using Email and Password
Sign up using Google
Sign up using Facebook
Sign up using Email and Password
Post as a guest
Required, but never shown
Required, but never shown
Required, but never shown
Required, but never shown
Required, but never shown
Required, but never shown
Required, but never shown
Required, but never shown
Required, but never shown
$begingroup$
I assume you are not counting multiplicity of the roots (i.e. the n roots are all distinct).
$endgroup$
– D.B.
Dec 25 '18 at 4:02
$begingroup$
@D.B. yes, even if there are duplicate roots, that's fine
$endgroup$
– CodeNoob
Dec 25 '18 at 4:04
1
$begingroup$
An example with $n=3$ is $x^3-x^2-x+1 = (x-1)^2(x+1)$. There don't seem to be any for $n=4, 5$, $6$ or $7$.
$endgroup$
– Robert Israel
Dec 25 '18 at 4:15
$begingroup$
@RobertIsrael that's right. We can prove all roots have abs value between (0.5,2) as well.
$endgroup$
– CodeNoob
Dec 25 '18 at 4:40
$begingroup$
Descartes’ Rule of Signs might be helpful here.
$endgroup$
– Clayton
Dec 25 '18 at 4:49